Đến nội dung

Hình ảnh

$(a-b)^2(b-c)^2(c-a)^2$

* * * * * 5 Bình chọn

  • Please log in to reply
Chủ đề này có 4 trả lời

#1
Nguyenhuyen_AG

Nguyenhuyen_AG

    Trung úy

  • Thành viên nổi bật 2016
  • 945 Bài viết

Gửi tặng đến các thành viên của VMF bài viết của mình cho kỷ yếu GGTH năm nay. Ban đầu bài này nằm chung một chuyên đề với Bổ đề hoán vị nhưng đọc lại thấy nó dài quá nên mình chia nó ra làm 3 phần: Bổ đề hoán vị, $(a-b)^2(b-c)^2(c-a)^2$ và còn một phần nữa vẫn đang viết.

 

Mọi người đọc bài viết và cho tác giả ý kiến nhé. :)

 

 

$(a-b)^2(b-c)^2(c-a)^2$

 

 

Trong bài viết nhỏ này tác giả xin được giới thiệu với bạn đọc những ứng dụng thú vị xoay quanh việc khai thác đại lượng $(a-b)^2(b-c)^2(c-a)^2$ kết hợp với kỹ thuật $pqr.$ Dù những lời giải và biến đổi trong bài đôi khi khá cồng kềnh nhưng hướng đi lại rất trực quan và hiệu quả. Hy vọng kỹ thuật này sẽ cung cấp cho bạn đọc có một số ý tưởng ban đầu trong lúc bối rối trước một bất đẳng thức khó, cũng như từ việc đánh giá và đẳng thức sẽ tìm được lời giải gọn và đẹp hơn.

 

Với mọi số thực $a,\,b,\,c$ bất kỳ ta luôn có
\begin{equation} \label{lab-1} % :v
P = (a-b)^2(b-c)^2(c-a)^2 \geqslant 0.
\end{equation}
Nếu đặt $p = a+b+c, \, q = ab+bc+ca$ và $r = abc$ (cách đặt này sẽ thống nhất cho cả bài viết) thì ta có thể biểu diễn
\begin{equation} \label{lab0}
P = p^2q^2 - 4q^3 + 2p(9q - 2p^2)r - 27r^2 = \frac{4(p^2-3q)^3-(2p^3-9pq+27r)^2}{27}.
\end{equation}
Đối với những ai từng nghiên cứu và sử dụng qua kỹ thuật $pqr,$ khai triển trên không quá xa lạ nhưng lại ít được sử dụng vì khá cồng kềnh. Bài viết này sẽ khai thác trực tiếp biến đổi trên và ứng dụng trong xử lý một số biểu thức hoán vị. Hy vọng qua đây, bạn đọc sẽ tìm thấy sự thú vị riêng cũng như nét độc đáo của bất đẳng thức.

 

1. Các bài toán mở đầu

Trong phần này, ta sẽ dùng kỹ thuật $pqr$ để trực tiếp xử lý những bất đẳng thức có chứa đại lượng $(a-b)(b-c)(c-a).$ Trước tiên, dùng khai triển \eqref{lab0} để chuyển bài toán về dạng $pqr.$ Sau đó, dựa trên những điều kiện của đề bài tìm ra mối liên hệ giữa $p,\,q$ và $r.$ Từ đó, có những đánh giá thích hợp hoặc dồn về một biến và khảo sát hàm. Để hiểu rõ hơn, mời bạn đọc cùng xem các bài toán sau đây.

Bài 1. Cho ba số thực $a,\,b,\,c$ và $t \geqslant 0$ cho trước thỏa mãn điều kiện $a+b+c=0,$ $a^2+b^2+c^2 = 6t^2.$ Tìm giá trị lớn nhất của biểu thức
\begin{equation} \label{lab1}
F = \left|(a-b)(b-c)(c-a)\right|.
\end{equation}

(Nguyễn Văn Huyện)

Lời giải. Từ giả thiết ta có
\[q = \frac{(a+b+c)^2 - (a^2+b^2+c^2)}{2} = -3t^2.\]
do đó $p = 0,\;q = -3t^2,$ suy ra
\[\begin{aligned}F^2 & = p^2q^2 - 4q^3 + 2p(9q - 2p^2)r - 27r^2 \\ & = 27(4t^6 - r^2) \leqslant 27 \cdot 4t^6 = 108t^6.\end{aligned}\]
Vậy $F \leqslant 6\sqrt{3}t^3$ đẳng thức xảy ra khi và chỉ khi
\[\left\{ \begin{aligned} & abc = 0 \\& a + b + c = 0 \\& ab + bc + ca = -3t^2 \end{aligned} \right.\]
Tức $a,\,b,\,c$ lần lượt là ba nghiệm của
\[x^3 - 3t^2x = 0.\]
Giải phương trình này ta được $(a, \, b, \, c) = (\sqrt{3}t, \, -\sqrt{3}t ,\, 0)$ cùng các hoán vị. Điều này cho phép ta kết luận $F_{\max} = 6\sqrt{3}t^3.$

Nhận xét. Trường hợp $t = 1$ ta được bài toán sau:

Cho ba số thực $a,\,b,\,c$ thỏa mãn $a+b+c=0$ và $a^2+b^2+c^2 = 6.$ Tìm giá trị lớn nhất của biểu thức
\[F = \left|(a-b)(b-c)(c-a)\right|.\]

(Turkey JBMO 2014)

Bài 2. Cho $a, \, b, \, c$ là ba số thực bất kỳ. Chứng minh rằng
\[{{(a-b)}^{3}}{{(b-c)}^{3}}+{{(b-c)}^{3}}{{(c-a)}^{3}}+{{(c-a)}^{3}}{{(a-b)}^{3}}+\frac{15}{4}{{(a-b)}^{2}}{{(b-c)}^{2}}{{(c-a)}^{2}} \leqslant 0.\]

(Liu Qian Bao, Nguyễn Văn Huyện)

Lời giải. Đặt $x=a-b,\,\,y=b-c,\,\,z=c-a$ thì $x+y+z=0$ bài toán trở thành
\begin{equation} \label{lab3}
{{x}^{3}}{{y}^{3}}+{{y}^{3}}{{z}^{3}}+{{z}^{3}}{{x}^{3}}+\frac{15}{4}{{x}^{2}}{{y}^{2}}{{z}^{2}} \leqslant 0.
\end{equation}

\[\begin{aligned} {{x}^{3}}{{y}^{3}}+{{y}^{3}}{{z}^{3}}+{{z}^{3}}{{x}^{3}} & ={{(xy+yz+zx)}^{3}}-3(xy+yz)(yz+zx)(zx+xy) \\& = {{(xy+yz+zx)}^{3}}+3{{x}^{2}}{{y}^{2}}{{z}^{2}} \\& = q^3+3r^2,\end{aligned}\]
nên \eqref{lab3} tương đương với
\[4q^3+27r^2 \leqslant 0.\]
Dễ thấy đây chính là \eqref{lab0} trong trường hợp $p = 0$ nên nó hiển nhiên đúng. Đẳng thức xảy ra khi và chỉ khi $a+b=2c,\,b+c=2a$ hoặc $c+a=2b.$ Bài toán được chứng minh.

Bài 3. Tìm hằng số $M$ nhỏ nhất sao cho bất đẳng thức
\[\left | ab(a^2-b^2)+bc(b^2-c^2)+ca(c^2-a^2) \right | \leqslant M(a^2+b^2+c^2)^2,\]
luôn đúng với mọi số thực $a,\,b,\,c$ thay đổi bất kỳ.

(Finbarr Holland, IMO 2006)

Lời giải. Ta chỉ cần xét $a^2+b^2+c^2>0,$ và thấy yêu cầu của bài toán đồng nghĩa với việc tìm giá trị lớn nhất của biểu thức
\[\begin{aligned} F & = \frac{\left|ab(a^2-b^2)+bc(b^2-c^2)+ca(c^2-a^2)\right|}{(a^2+b^2+c^2)^2} \\& = \frac{\left | (a+b+c)(a-b)(b-c)(c-a)\right |}{(a^2+b^2+c^2)^2}. \end{aligned}\]
Nếu thay $(a,b,c)$ bởi $(-a,-b,-c)$ thì bài toán vẫn không đổi nên ta có thể giả sử $a+b+c>0$ và chuẩn hóa $a+b+c=1,$ khi đó
\[F = \frac{\left|(a-b)(b-c)(c-a)\right|}{(1-2q)^2} = \frac{\sqrt{\frac{4(1 - 3q)^3 - [27r - (9q-2)]^2}{27}}}{(1-2q)^2} \leqslant \frac{2(1 - 3q)\sqrt{3(1 - 3q)}}{9(1 - 2q)^2}.\]
Đặt
\[f(q) = \frac{2(1 - 3q)\sqrt{3(1 - 3q)}}{9(1 - 2q)^2},\]
thì
\[f^{'}(q) = \frac{(1+6q)(3q-1)}{3(2q-1)^3\sqrt{3(1-3q)}},\]
do đó phương trình $f^{'}(q) = 0$ có nghiệm duy nhất $q = -\frac{1}{6}.$ Lập bảng biến thiên ta thấy
\[f(q) \leqslant f\left(-\frac{1}{6}\right)= \frac{9\sqrt{2}}{32}.\]
Do đó $F \leqslant \frac{9\sqrt{2}}{32},$ đẳng thức xảy ra khi $p = 1,\,q = - \frac{1}{6},\,r = - \frac{7}{54},$ tức $a,\,b,\,c$ lần lượt là ba nghiệm của
\[t^3 - t^2 - \frac{1}{6}t + \frac{7}{54} = 0.\]
Từ đó ta được
\[a = \frac{2-3\sqrt{2}}{2} \cdot c , \; b = \frac{2+3\sqrt{2}}{2} \cdot c,\]
hoặc
\[a = \frac{-11+6\sqrt{2}}{7} \cdot c, \; b = \frac{-2+3\sqrt{2}}{7} \cdot c.\]
Vậy giá trị nhỏ nhất cần tìm là $M = \frac{9\sqrt{2}}{32}.$

Bài 4. Tìm hằng số $k$ lớn nhất sao cho bất đẳng thức
\begin{equation} \label{80}
\frac{a^2+b^2+c^2}{ab+bc+ca} - 1 \geqslant k\left ( \frac{a-b}{a+b}+\frac{b-c}{b+c}+\frac{c-a}{c+a} \right )^2,
\end{equation}
luôn đúng với mọi số thực $a,\,b,\,c$ không âm thỏa mãn $ab+bc+ca>0.$

(Nguyễn Văn Huyện)

Lời giải. Với $a = 2 + \sqrt{3},\,b=1,\,c=0$ thì \eqref{80} trở thành $k \leqslant 9.$ Ta sẽ chứng minh $k = 9$ là giá trị lớn nhất cần tìm, tức chứng minh
\[\frac{a^2+b^2+c^2}{ab+bc+ca} - 1 \geqslant 9\left ( \frac{a-b}{a+b}+\frac{b-c}{b+c}+\frac{c-a}{c+a} \right )^2,\]
hay là
\[\frac{a^2+b^2+c^2}{ab+bc+ca} - 1 \geqslant \frac{9(a-b)^2(b-c)^2(c-a)^2}{(a+b)^2(b+c)^2(c+a)^2}.\]
Đổi biến về $pqr$ như sau
\[\frac{p^2-3q}{q} \geqslant \frac{9\left[p^2q^2-4q^3+2p(9q-2p^2)r-27r^2\right]}{(pq-r)^2},\]
hoặc
\begin{equation} \label{lab8}
(p^2-3q)(pq-r)^2 \geqslant 9q\left[p^2q^2-4q^3+2p(9q-2p^2)r-27r^2\right].
\end{equation}
Chuẩn hóa $p=1$ và nhóm lại theo $r$ bất đẳng thức \eqref{lab8} trở thành
\[(1 + 240q)r^2 + 2q(17 - 78q)r + q^2(1 - 6q)^2 \geqslant 0.\]
Nếu $0 \leqslant q \leqslant \frac{17}{78}$ thì bất đẳng thức hiển nhiên đúng. Còn nếu $\frac{17}{78} \leqslant q \leqslant  \frac{1}{3}$ đặt
\[f(r) = (1 + 240q)r^2 + 2q(17 - 78q)r + q^2(1 - 6q)^2.\]
Ta có
\[\begin{aligned}\Delta_{f(r)}^{'} & = [q(17-78q)]^2-(1+240q)q^2(1-6q)^2 \\& = - 288q^2(1 - 3q)(1 - 2q)(5q - 1). \end{aligned}\]
Dễ thấy $1 \geqslant 3q \geqslant 2q$ và
\[5q - 1 \geqslant 5\cdot \frac{17}{78} - 1 = \frac{7}{78} > 0.\]
Cho nên $\Delta_{f(r)}^{'} \leqslant 0$ suy ra $f(r) \geqslant 0.$ Đẳng thức xảy ra khi $a=b=c$ hoặc $\frac{a}{b} = 2 \pm \sqrt{3} ,\,c=0$ cùng các hoán vị.

Bài 5. Với $a,\,b,\,c$ là ba số thực dương và số thực $k \geqslant 0$ cho trước thỏa mãn điều kiện
\[(a+b+c)\left ( \frac{1}{a}+\frac{1}{b}+\frac{1}{c} \right )=(k+3)^2.\]
Chứng minh rằng
\[\left |\frac{(a-b)(b-c)(c-a)}{abc}  \right | \leqslant \sqrt{k^3(k+4)}.\]

(Nguyễn Văn Huyện)

Lời giải. Chú ý rằng
\begin{equation} \label{lab7}
\begin{aligned}\left |\frac{(a-b)(b-c)(c-a)}{abc}  \right | & = \sqrt{\frac{(a-b)^2(b-c)^2(c-a)^2}{a^2b^2c^2}} \\& = \sqrt{\frac{p^2q^2 - 4q^3 + 2p(9q - 2p^2)r - 27r^2}{r^2}}.\end{aligned}
\end{equation}
Từ giả thiết ta có $\displaystyle p = \frac{r(k+3)^2}{q},$ thay giá trị này vào \eqref{lab7} và biến đổi, ta được
\[\left |\frac{(a-b)(b-c)(c-a)}{abc}  \right | = \sqrt{(k+3)^4+18(k+3)^2-27-4\left[\frac{(k+3)^6r^2}{q^3}+\frac{q^3}{r^2} \right]}.\]
Theo bất đẳng thức AM-GM thì
\[\frac{(k+3)^6r^2}{q^3}+\frac{q^3}{r^2} \geqslant 2(k+3)^3,\]
do đó
\[\left |\frac{(a-b)(b-c)(c-a)}{abc}  \right | \leqslant \sqrt{(k+3)^4+18(k+3)^2-27-8(k+3)^3} = \sqrt{k^3(k+4)}.\]
Đẳng thức xảy ra khi chẳng hạn
\[\frac{a}{c} = \frac{k+2+\sqrt{k(k+4)}}{2},\,\frac{b}{c} = \frac{k+2-\sqrt{k(k+4)}}{2}.\]
Chứng minh hoàn tất.

Bài 6. Chứng minh rằng
\begin{equation} \label{lab5}
(x^2-x+1)(y^2-y+1)(z^2-z+1) \geqslant (x-y)(y-z)(z-x),
\end{equation}
trong đó $x,\,y,\,z$ là ba số thực thay đổi bất kỳ.

(Ji Mun Kwon)

Lời giải. Thay $(x,y,z)$ bởi $\left ( \frac{1+a}{2},\frac{1+b}{2},\frac{1+c}{2} \right )$ bất đẳng thức \eqref{lab5} trở thành
\[(a^2+3)(b^2+3)(c^2+3) \geqslant 8(a-b)(b-c)(c-a).\]
Theo bất đẳng thức AM-GM thì
\[\frac{q^2}{3} + 6q + 27 \geqslant 0,\]

\[p^2 + 2pr + r^2 \geqslant 0.\]
Do đó
\[\begin{aligned}(a^2+3)(b^2+3)(c^2+3) & =a^2b^2c^2+27+3(a^2b^2+b^2c^2+c^2a^2)+9(a^2+b^2+c^2) \\&= r^2+27+3(3p^2+q^2-2pr-6q) \\ & \geqslant -(p^2+2pr) -\left(\frac{q^2}{3} + 6q\right)+ 3(3p^2+q^2-2pr-6q) \\ &=\frac{8}{3}\left[3(p^2-3q) + (q^2-3pr) \right]\\ & \geqslant \frac{16}{3} \sqrt{3(p^2-3q)(q^2-3pr)}.\end{aligned}\]
Như vậy ta cần chỉ ra
\[4(p^2-3q)(q^2-3pr) \geqslant 3(a-b)^2(b-c)^2(c-a)^2,\]
tương đương với
\[4(p^2-3q)(q^2-3pr) \geqslant 3\big[p^2q^2-4q^3+2p(9q-2p^2)r-27r^2\big],\]
thu gọn thành
\[(pq-9r)^2 \geqslant 0.\]
Đẳng thức xảy ra khi và chỉ khi $y = \frac{z-1}{z},\,x = \frac{1}{1-z},\,z \ne 0,\,z \ne 1$ cùng các hoán vị. Chứng minh hoàn tất.

Nhận xét. Lời giải dựa vào ý tưởng của anh Võ Quốc Bá Cẩn trên diễn đàn AoPS.

Bài 7. Cho ba số thực $a,\,b,\,c$ thỏa mãn đồng thời các điều kiện
\[a+b+c=0, \; a^2+b^2+c^2=3.\]
Chứng minh rằng
\begin{equation} \label{41}
-\frac{9}{2} \leqslant a^5b+b^5c+c^5a \leqslant -3.
\end{equation}

(Nguyễn Văn Huyện, Lê Việt Hải)

Lời giải. Từ giả thiết ta có
\[\left\{\begin{aligned} & ab+bc+ca = - \frac{3}{2} \\& a^2b^2+b^2c^2+c^2a^2 = \frac{9}{4} \\ & a^3+b^3+c^3 = 3abc \end{aligned}\right.\]

\[\begin{aligned}a^5b+b^5c+c^5a & = (a+b+c) \sum a^4b - \left(a^4b^2+b^4c^2+c^4a^2 + abc \sum a^3\right) \\& = -(a^4b^2+b^4c^2+c^4a^2+3a^2b^2c^2).\end{aligned}\]
nên ta có thể viết \eqref{41} lại dưới dạng
\begin{equation} \label{lab4}
3 \leqslant a^4b^2+b^4c^2+c^4a^2+3a^2b^2c^2 \leqslant \frac{9}{2}.
\end{equation}
Lại có
\[\begin{aligned} 2(a^4b^2+b^4c^2+c^4a^2) & = \sum a^2 \sum b^2c^2 - 3a^2b^2c^2 - \prod (a^2-b^2) \\ & =\frac{27}{4} - 3a^2b^2c^2 - \prod (a^2-b^2),\end{aligned}\]
nên \eqref{lab4} tương đương với
\[3a^2b^2c^2 - \frac{9}{4} \leqslant (a^2 - b^2)(b^2 - c^2)(c^2 - a^2) \leqslant 3a^2b^2c^2 + \frac{3}{4}.\]
hay
\[3r^2 - \frac{9}{4} \leqslant (a^2 - b^2)(b^2 - c^2)(c^2 - a^2) \leqslant 3r^2 + \frac{3}{4}.\]
Ta chứng minh
\[(a^2 - b^2)(b^2 - c^2)(c^2 - a^2) \leqslant 3r^2 + \frac{3}{4}.\]
Thật vậy, vì
\[\begin{aligned}(a^2 - b^2)(b^2 - c^2)(c^2 - a^2) & \leqslant \left|(a^2-b^2)(b^2-c^2)(c^2-a^2)\right| \\& = \sqrt{(a+b)^2(b+c)^2(c+a)^2\cdot(a-b)^2(b-c)^2(c-a)^2}  \\ & = \sqrt{(pq-r)^2\left[p^2q^2-4q^3+2p(9q-2p^2)r-27r^2\right]} \\& = \sqrt{\frac{27r^2(1 - 2r^2)}{2}},\end{aligned}\]

\[\frac{27r^2(1 - 2r^2)}{2} - \left(3r^2 + \frac{3}{4}\right)^2 = -\frac{9}{16}(8r^2 - 1)^2 \leqslant  0,\]
nên bất đẳng thức trên đúng. Đẳng thức xảy ra khi và chỉ khi $r^2 = \frac{1}{8},\,p = 0$ và $q = -\frac{3}{2}$ tức $a, \, b, \, c$ là ba nghiệm của
\[\left\{ \begin{aligned} & \left[ \begin{aligned} & {{t}^{3}}-\frac{3}{2}t-\frac{1}{\sqrt{8}}=0 \\& {{t}^{3}}-\frac{3}{2}t+\frac{1}{\sqrt{8}}=0\end{aligned} \right. \\& ({{a}^{2}}-{{b}^{2}})({{b}^{2}}-{{c}^{2}})({{c}^{2}}-{{a}^{2}}) \geqslant 0
\end{aligned} \right.\]
Giải hệ phương trình này ta được $(a, \, b, \, c)$ là một hoán vị của một trong hai bộ ba dưới đây
\[\left\{\left(\sqrt{2}\cos \frac{4\pi}{9},\,\sqrt{2}\cos \frac{2\pi}{9},\,-\sqrt{2}\cos \frac{\pi}{9}\right),\,\left(-\sqrt{2}\cos \frac{4\pi}{9},\,-\sqrt{2}\cos \frac{2\pi}{9},\,\sqrt{2}\cos \frac{\pi}{9}\right)\right\}.\]
Tiếp đến ta chứng minh
\[(a^2 - b^2)(b^2 - c^2)(c^2 - a^2) \geqslant - \frac{9}{4} + 3r^2.\]
Đánh giá tương tự, ta có
\[(a^2 - b^2)(b^2 - c^2)(c^2 - a^2) \geqslant - \sqrt{\frac{27r^2(1 - 2r^2)}{2}}.\]
Do đó ta chỉ cần chỉ ra
\[\sqrt{\frac{27r^2(1 - 2r^2)}{2}} \leqslant \frac{9}{4} - 3r^2.\]
Chú ý rằng $r^2 \leqslant \frac{1}{2}$ cho nên bất đẳng thức này đúng vì
\[\frac{27r^2(1 - 2r^2)}{2} - \left (\frac{9}{4} - 3r^2  \right )^2 = - \frac{9}{16}(8r^2-3)^2 \leqslant 0.\]
Đẳng thức xảy ra khi và chỉ khi $r^2 = \frac{3}{8},\,p = 0$ và $q = -\frac{3}{2}$ tức $a, \, b, \, c$ là ba nghiệm của
\[\left\{ \begin{aligned} & \left[ \begin{aligned} & {{t}^{3}}-\frac{3}{2}t-\sqrt{\frac{3}{8}}=0 \\& {{t}^{3}}-\frac{3}{2}t+\sqrt{\frac{3}{8}}=0\end{aligned} \right. \\& ({{a}^{2}}-{{b}^{2}})({{b}^{2}}-{{c}^{2}})({{c}^{2}}-{{a}^{2}}) \leqslant 0
\end{aligned} \right.\]
Giải hệ phương trình này ta được $(a, \, b, \, c)$ là một hoán vị của một trong hai bộ ba dưới đây
\[\left\{\left (\sqrt{2}\cos \frac{\pi}{18},\,-\sqrt{2}\sin \frac{2\pi}{9},\,-\sqrt{2}\sin \frac{\pi}{9}\right ),\,\left (-\sqrt{2}\cos \frac{\pi}{18},\,\sqrt{2}\sin \frac{2\pi}{9},\,\sqrt{2}\sin \frac{\pi}{9}\right )\right\}.\]
Bài toán được chứng minh.


Bài viết đã được chỉnh sửa nội dung bởi Nguyenhuyen_AG: 29-07-2016 - 13:54

Nguyen Van Huyen
Ho Chi Minh City University Of Transport

#2
Nguyenhuyen_AG

Nguyenhuyen_AG

    Trung úy

  • Thành viên nổi bật 2016
  • 945 Bài viết

2. Bất đẳng thức hoán vị

Sang phần này, ta sẽ tiếp tục khai thác đại lượng $P$ trong việc xử lý các bất đẳng thức hoán vị. Đây là một hướng phát triển khác của $pqr$ được gọi là “$pqr$ hoán vị” và được thực hiện như sau. Biến đổi khéo léo các biểu thức hoán vị để làm xuất hiện $Q=(a-b)(b-c)(c-a)$ cũng là một đại lương hoán vị. Tuy nhiên, ta hoàn toàn có thể biễu diễn $Q$ dưới dạng $pqr$ thông qua đánh giá $Q \leqslant |Q| = \sqrt{P}$ đến đây, sử dụng \eqref{lab0} và ta giải tiếp bằng $pqr.$ Lưu ý rằng, ngoài các quan hệ sẵn có, bản thân bất đẳng thức $P \geqslant 0$ cũng đã cho ta một đánh giá tương quan giữa $p,\,q$ và $r.$

Bài 8. Với $a,\,b,\,c$ là các số thực và $t \geqslant 0$ cho trước thỏa mãn điều kiện $a+b+c=0$ và $a^2+b^2+c^2=6t^2.$ Chứng minh rằng với mọi số thực $k$ ta luôn có
\[a^2b+b^2c+c^2a+kabc \leqslant 2t^3\sqrt{k^2 - 3k + 9}.\]

(Võ Quốc Bá Cẩn)


Lời giải. Chú ý rằng với mọi số thực $x$ ta luôn có $\max\{- x,\,x\} \leqslant \sqrt{x^2},$ do đó
\[\begin{aligned} 2(a^2b+b^2c+c^2a) & = \sum ab(a+b) - (a-b)(b-c)(c-a) \\ &= pq - 3r - (a-b)(b-c)(c-a) \\& \leqslant pq - 3r + \sqrt{(a-b)^2(b-c)^2(c-a)^2}.\end{aligned}\]
Cho nên
\[a^2b+b^2c+c^2a \leqslant  \frac{pq - 3r + \sqrt{(a-b)^2(b-c)^2(c-a)^2}}{2},\]
hay là
\begin{equation} \label{lab9}
a^2b+b^2c+c^2a \leqslant \frac{pq - 3r + \sqrt{p^2q^2 - 4q^3 + 2p(9q - 2p^2)r - 27r^2}}{2}.
\end{equation}
Từ giả thiết dễ thấy $p = 0, \, q = -3t^2,$ áp dụng \eqref{lab9} ta có
\[\begin{aligned}a^2b+b^2c+c^2a + kabc & \leqslant \frac{pq - 3r + \sqrt{p^2q^2 - 4q^3 + 2p(9q - 2p^2)r - 27r^2}}{2} + kr \\ &  = \left(k - \frac{3}{2}\right)r + \frac{3}{2}\sqrt{12t^6 - 3r^2}.\end{aligned}\]
Xét hàm số
\[f(r)  = \left(k - \frac{3}{2}\right)r + \frac{3}{2}\sqrt{12t^6 - 3r^2}.\]
Tính đạo hàm
\[f^{'}(r) = k - \frac{3}{2} - \frac{9r}{2\sqrt{12t^6-3r^2}}.\]
Do đó phương trình $f^{'}(r) = 0$ có nghiệm
\[r = r_0 = \frac{t^3(2k-3)}{\sqrt{k^2-3k+9}}\]
Lập bảng biến thiên ta thấy
\[\begin{aligned} a^2b+b^2c+c^2a + kabc & \leqslant f(r_0) = \left(k - \frac{3}{2}\right)r_0 + \frac{3}{2}\sqrt{12t^6 - 3r_0^2} \\ & = \left(k - \frac{3}{2}\right)\cdot\frac{t^3(2k-3)}{\sqrt{k^2-3k+9}} + \frac{3}{2}\sqrt{12t^6 - \frac{3t^6(2k-3)^2}{k^2-3k+9}} \\ & = 2t^3\sqrt{k^2 - 3k + 9}.\end{aligned}\]
Như vậy
\[a^2b+b^2c+c^2a+kabc \leqslant 2t^3\sqrt{k^2 - 3k + 9}.\]
Đẳng thức xảy ra khi và chỉ khi
\[\left\{\begin{aligned} & a + b + c = 0 \\ & ab + bc + ca = -3t^2 \\ &  abc = \pm \frac{t^3(2k - 3)}{\sqrt{k^2 - 3k + 9}} \\ & (a-b)(b-c)(c-a) \leqslant 0\end{aligned}\right.\]
Bài toán được chứng minh.

Nhận xét. Trường hợp $k=0,\,t=1$ ta được bài toán sau:

Cho ba số thực $a,\,b,\,c$ thỏa mãn điều kiện $a+b+c=0$ và $a^2+b^2+c^2=6.$ Chứng minh rằng
\[a^2b+b^2c+c^2a \leqslant 6.\]

(British MO 1986)

Bài 9. Cho ba số thực $a,\,b,\,c$ thỏa mãn điều kiện $a^2+b^2+c^2>0.$ Chứng minh rằng
\[ - \frac{\sqrt{7}}{8} \leqslant \frac{a^3b+b^3c+c^3a}{(a^2+b^2+c^2)^2} \leqslant \frac{1}{3}.\]

(Vasile Cîrtoaje, Dan Chen)

Lời giải. Ta có hai bổ đề sau
\begin{equation} \label{lab14}
a^3b+b^3c+c^3a \leqslant \frac{p^4+9p^2q-27q^2+2\sqrt{7p^2(p^2-3q)^3}}{27},
\end{equation}

\begin{equation} \label{lab15}
a^3b+b^3c+c^3a \geqslant \frac{p^4+9p^2q-27q^2-2\sqrt{7p^2(p^2-3q)^3}}{27}.
\end{equation}
Chú ý rằng
\[\begin{aligned} 2(a^3b+b^3c+c^3a) & = \sum ab(a^2+b^2) - (a+b+c)(a-b)(b-c)(c-a) \\& =  p^2q-2q^2-pr+\sqrt{(a+b+c)^2(a-b)^2(b-c)^2(c-a)^2}.\end{aligned}\]
Cho nên
\[a^3b+b^3c+c^3a \leqslant \frac{p^2q-pr-2q^2+\sqrt{p^2[p^2q^2 - 4q^3 + 2p(9q - 2p^2)r - 27r^2]}}{2}.\]
Xét hàm số
\[f(r) = \frac{p^2q-pr-2q^2+\sqrt{p^2[p^2q^2 - 4q^3 + 2p(9q - 2p^2)r - 27r^2]}}{2},\]
tính đạo hàm ta được
\[f^{'}(r) = \frac{p^2[p(9q-2p^2)-27r]}{2\sqrt{p^2[p^2q^2-4q^3+2p(9q-2p^2)r-27r^2]}}-\frac{p}{2}.\]
Do đó phương trình $f^{'}(r)$ có nghiệm
\[\left[\begin{aligned}r_0 & = \frac{14p^3+63pq-\sqrt{7(p^2-3q)^3}}{189} \\ r_1 & = \frac{14p^3+63pq+\sqrt{7(p^2-3q)^3}}{189} \end{aligned}\right.\]
Lập bảng biến thiên ta thấy
\[f(r) \leqslant f(r_0) = \frac{p^4+9p^2q-27q^2+2\sqrt{7p^2(p^2-3q)^3}}{27}.\]
Bất đẳng thức \eqref{lab15} chứng minh tương tự. Quay trở lại bài toán, ta sẽ chứng minh
\[(a^2+b^2+c^2)^2 \geqslant 3(a^3b+b^3c+c^3a).\]
Áp dụng \eqref{lab14} ta cần chỉ ra
\begin{equation} \label{lab17}
9(p^2-2q)^2 \geqslant p^4+9p^2q-27q^2+2\sqrt{7p^2(p^2-3q)^3}.
\end{equation}
Đặt $p^2 = kq$ bất đẳng thức \eqref{lab17} trở thành
\[9\left(p^2-2\cdot \frac{p^2}{k}\right)^2 \geqslant p^4+9p^2\cdot \frac{p^2}{k}-27\left (\frac{p^2}{k} \right )^2+2\sqrt{7p^2\left(p^2-3\cdot \frac{p^2}{k}\right)^3},\]
tương đương với
\[8k^2-45k+63 \geqslant 2\sqrt{7k(k-3)^2},\]
thu gọn thành
\[9(k-3)^2(2k-7)^2 \geqslant 0.\]
Hiển nhiên đúng, đẳng thức xảy ra khi và chỉ khi $a=b=c$ hoặc \[\frac{a}{\sin^2\frac{4\pi}{7}} = \frac{b}{\sin^2\frac{2\pi}{7}} = \frac{c}{\sin^2\frac{\pi}{7}}.\]
Tiếp đến ta chứng minh
\[(a^2+b^2+c^2)^2+\frac{8}{\sqrt{7}}(a^3b+b^3c+c^3a) \geqslant 0.\]
Áp dụng \eqref{lab15} ta cần chỉ ra được
\begin{equation} \label{lab18}
(p^2-2q)^2 + \frac{8}{\sqrt{7}} \cdot \frac{p^4+9p^2q-27q^2-2\sqrt{7p^2(p^2-3q)^3}}{27} \geqslant 0.
\end{equation}
Đặt $p^2 = kq$ bất đẳng thức \eqref{lab18} trở thành
\[\left(p^2-2\cdot \frac{p^2}{k}\right)^2 + \frac{8}{\sqrt{7}} \cdot \frac{p^4+9p^2\cdot \frac{p^2}{k}-27\left (\frac{p^2}{k} \right )^2-2\sqrt{7p^2\left(p^2-3\cdot \frac{p^2}{k}\right)^3}}{27} \geqslant 0,\]
hay là
\begin{equation} \label{lab19}
27\sqrt{7}(k-2)^2+8(k^2+9k-27) \geqslant 16k(k-3)\sqrt{\frac{7(k-3)}{k}}.
\end{equation}
Đặt $x = \sqrt{\frac{k-3}{k}} > 0,$ thì $k = \frac{3}{1-x^2}$ thay giá trị này vào \eqref{lab19} và thu gọn lại ta được
\[\frac{3\big(\sqrt{7}-2\big)\left[18x^2+2\big(\sqrt{7}-1\big)x+2+\sqrt{7}\right]\big(2x-3-\sqrt{7}\big)^2}{2(x^2-1)^2} \geqslant 0.\]
Từ đó suy ra điều phải chứng minh.

 

Bài 10. Cho ba số thực không âm $a,\,b,\,c$ thỏa mãn $a+b+c = 3.$ Chứng minh rằng
\[a^2b+b^2c+c^2a+abc\left[1+\frac{1}{6}(a^2+b^2+c^2-ab-bc-ca)\right]\leqslant 4.\]

(Võ Quốc Bá Cẩn)

Lời giải. Trước hết ta có
\begin{equation} \label{lab10}
r \leqslant \frac{q^2(p^2-q)}{2p(2p^2-3q)}.
\end{equation}
Áp dụng đánh giá
\[r \leqslant \frac{p(9q-2p^2)+2\sqrt{(p^2-3q)^3}}{27}, \; \text{ (xem ở \eqref{lab23}),}\]
và bất đẳng thức AM-GM, ta có
\[\begin{aligned} 27r & \leqslant p(9q-2p^2) + \frac{(p^2-3q)\cdot 2 \cdot \big(p^2-\frac{3}{2}q\big)\cdot p\sqrt{p^2-3q}}{p\left(p^2-\frac{3}{2}q\right)^2} \\& \leqslant p(9q-2p^2) + \frac{(p^2-3q)\left[\left(p^2-\frac{3}{2}q\right)^2 + p^2(p^2-3q)\right]}{p\left(p^2-\frac{3}{2}q\right)} \\& = \frac{27q^2(p^2-q)}{2p(2p^2-3q)}.\end{aligned}\]
Tức \eqref{lab10} đúng. Quay trở lại bài toán, áp dụng \eqref{lab9} ta cần chứng minh
\[\frac{pq - 3r + \sqrt{p^2q^2 - 4q^3 + 2p(9q - 2p^2)r - 27r^2}}{2}+r\left[1+\frac{1}{6}(p^2-3q)\right]\leqslant 4,\]
hay là
\begin{equation} \label{baitoan15}
\sqrt{p^2q^2 - 4q^3 + 2p(9q - 2p^2)r - 27r^2} \leqslant 24 -3(2 - q)r - 9q.
\end{equation}
Ta sẽ chỉ ra
\[3(2 - q)r + 9q \leqslant 24.\]
Thật vậy, vì $p = 3$ nên từ \eqref{lab10} ta được
\[r \leqslant \frac{q^2(p^2-q)}{2p(2p^2-3q)} = \frac{q^2(9-q)}{18(6-q)},\]
do đó
\[3(2 - q)r + 9q \leqslant \frac{q^2(2 - q)(9-q)}{6(6 - q)} + 9q.\]
Mặt khác ta lại có
\[\frac{q^2(2 - q)(9-q)}{6(6 - q)} + 9q - 24 = \frac{(q-3)[q^3 + 36 + 4(3 - q)(2q + 21)]}{6(6 - q)} \leqslant 0,\]
cho nên
\[3(2 - q)r + 9q \leqslant 24.\]
Như vậy vế phải của \eqref{baitoan15} là một số không âm, kết hợp với $p = 3$ ta bình phương hai vế và nhóm theo $r$ bất đẳng thức \eqref{baitoan15} trở thành
\[[27 + (2 - q)^2]r^2 + 2(3q + 19)(2 - q)r + 4(q + 4)(2 - q)^2 \geqslant 0.\]
Đặt
\[f(r) = [27 + (2 - q)^2]r^2 + 2(3q + 19)(2 - q)r + 4(q + 4)(2 - q)^2,\]
ta có
\[\begin{aligned}\Delta^{'}_{f(r)} & = (3q + 19)^2(2 - q)^2 - 4[27 + (2 - q)^2](q + 4)(2 - q)^2 \\& = -4(4q + 15)(2 - q)^2(3 - q)^2 \leqslant 0.\end{aligned}\]
Do đó $f(r) \geqslant 0,$ đẳng thức xảy ra khi và chỉ khi $a = b = c = 1$ hoặc $a = 0,\,b = 1,\,c=2$ cùng các hoán vị. Bài toán được chứng minh.


Bài viết đã được chỉnh sửa nội dung bởi Nguyenhuyen_AG: 27-07-2016 - 22:37

Nguyen Van Huyen
Ho Chi Minh City University Of Transport

#3
Nguyenhuyen_AG

Nguyenhuyen_AG

    Trung úy

  • Thành viên nổi bật 2016
  • 945 Bài viết

3. Các bài toán chọn lọc

 

Bài 11. Cho ba số thực không âm $a,\,b,\,c$ thay đổi thỏa mãn $a+b+c>0$ và $k \ne 0$ cho trước sao cho
\[a^2+b^2+c^2 = \left (1+\frac{3}{k^2-1}  \right )(ab+bc+ca).\]
Chứng minh rằng
\begin{equation} \label{lab22}
\frac{(k-2)(k+1)^2}{27k^3} \leqslant \frac{abc}{(a+b+c)^3} \leqslant \frac{(k+2)(k-1)^2}{27k^3}
\end{equation}

(Nguyễn Văn Huyện)

Lời giải. Do
\[ p^2q^2 - 4q^3 + 2p(9q - 2p^2)r - 27r^2 \geqslant 0,\]
là một tam thức bậc hai theo $r$ và tam thức này có
\[\Delta^{'} = \left[p(9q-2p^2)\right]^2+27(p^2q^2-4q^3) = 4(p^2-3q)^3 \geqslant 0,\]
cho nên
\begin{equation} \label{lab23}
\frac{p(9q-2p^2)-2\sqrt{(p^2-3q)^3}}{27} \leqslant r \leqslant \frac{p(9q-2p^2)+2\sqrt{(p^2-3q)^3}}{27}.
\end{equation}
Viết giả thiết bài toán lại dưới dạng
\[(a+b+c)^2 = \frac{3k^2}{k^2-1}(ab+bc+ca),\]
tức $q = \displaystyle \frac{(k^2-1)p^2}{3k^2}$ thay các giá trị này vào \eqref{lab23}, ta được
\[r \leqslant \frac{p\left[9 \cdot \frac{(k^2-1)p^2}{3k^2} -2p^2\right]+2\sqrt{\left[p^2-3 \cdot \frac{(k^2-1)p^2}{3k^2}\right]^3}}{27} = \frac{p^3(k+2)(k-1)^2}{27k^3}.\]
Đẳng thức xảy ra khi và chỉ khi $\left(1-\frac{3}{k+2}\right)a=b=c.$ Tương tự
\[r \geqslant \frac{p\left[9 \cdot \frac{(k^2-1)p^2}{3k^2} -2p^2\right]-2\sqrt{\left[p^2-3 \cdot \frac{(k^2-1)p^2}{3k^2}\right]^3}}{27} = \frac{p^3(k-2)(k+1)^2}{27k^3}.\]
Đẳng thức xảy ra khi và chỉ khi $\left(1+\frac{3}{k+2}\right)a=b=c.$ Vậy
\[\frac{(k-2)(k+1)^2}{27k^3} \leqslant \frac{r}{p^3} \leqslant  \frac{(k+2)(k-1)^2}{27k^3}.\]
Bài toán được chứng minh.

 

Nhận xét. Trường hợp $k=2$ ta được bài toán sau:

Cho ba số thực dương $a,\,b,\,c$ thỏa mãn điều kiện $a^2+b^2+c^2=2(ab+bc+ca).$ Chứng minh rằng
\[\frac{a+b+c}{\sqrt[3]{abc}} \geqslant 3\sqrt[3]{2}.\]

(Michael Rozenberg, Iran MO 2014)

Bài 12. Với $a,\,b,\,c$ là ba số thực dương và $k$ là số thực cho trước thỏa mãn điều kiện
\[a^2+b^2+c^2 = \left (1+\frac{3}{k^2-1}  \right )(ab+bc+ca).\]
(a) Chứng minh rằng nếu $k > 1$ thì
\[\frac{a}{b+c}+\frac{b}{c+a}+\frac{c}{a+b} \leqslant \frac{3(2k^2-k+2)}{2(2k+1)(k-1)}.\]
(b) Chứng minh rằng nếu $k > 2$ thì
\[\frac{a}{b+c}+\frac{b}{c+a}+\frac{c}{a+b} \geqslant \frac{3(2k^2+k+2)}{2(k+1)(2k-1)}.\]

(Nguyễn Văn Huyện)

Lời giải. (a) Tương tự như trên ta có $q = \displaystyle \frac{(k^2-1)p^2}{3k^2}$ và
\[0 < r \leqslant \frac{p^3(k+2)(k-1)^2}{27k^3},\]
cho nên
\[\begin{aligned}\frac{a}{b+c}+\frac{b}{c+a}+\frac{c}{a+b} = \frac{p(p^2+q)}{pq-r}-3 &\leqslant \frac{p\left[p^2+\frac{(k^2-1)p^2}{3k^2}\right]}{p\cdot \frac{(k^2-1)p^2}{3k^2}-\frac{p^3(k+2)(k-1)^2}{27k^3}}-3\\&=\frac{3(2k^2-k+2)}{2(2k+1)(k-1)}. \end{aligned}\]
Đẳng thức xảy ra khi và chỉ khi $\left(1-\frac{3}{k+2}\right)a=b=c.$

(b) Tương tự do $k > 2$ nên $\displaystyle \frac{(k-2)(k+1)^2}{27k^3} > 0,$ do đó
\[\begin{aligned}\frac{a}{b+c}+\frac{b}{c+a}+\frac{c}{a+b} = \frac{p(p^2+q)}{pq-r}-3 &\geqslant \frac{p\left[p^2+\frac{(k^2-1)p^2}{3k^2}\right]}{p\cdot \frac{(k^2-1)p^2}{3k^2}-\frac{p^3(k-2)(k+1)^2}{27k^3}}-3\\&=\frac{3(2k^2+k+2)}{2(k+1)(2k-1)}. \end{aligned}\]
Đẳng thức xảy ra khi và chỉ khi $\left(1+\frac{3}{k+2}\right)a=b=c.$ Chứng minh hoàn tất.
[/proof]

Nhận xét. Trường hợp $k = \frac{5}{2}$ ta được bài toán sau

Cho ba số dương $a,\,b,\,c$ thỏa mãn điều kiện $a^2+b^2+c^2 = \frac{11}{7}(ab+bc+ca).$ Chứng minh rằng
\[\frac{51}{28} \leqslant \frac{a}{b+c}+\frac{b}{c+a}+\frac{c}{a+b} \leqslant 2.\]

(Vasile Cîrtoaje)

Bài 13. Với $a,\,b,\,c$ là ba số thực dương và $k$ là số thực cho trước thỏa mãn điều kiện
\[a^2+b^2+c^2 = \left (1+\frac{3}{k^2-1}  \right )(ab+bc+ca).\]
(a) Chứng minh rằng nếu $k > 2$ thì
\[\frac{a+b}{c}+\frac{b+c}{a}+\frac{c+a}{b} \leqslant \frac{6(k^2-k+1)}{(k+1)(k-2)}.\]
(b) Chứng minh rằng nếu $k > 1$ thì
\[\frac{a+b}{c}+\frac{b+c}{a}+\frac{c+a}{b} \geqslant \frac{6(k^2+k+1)}{(k+2)(k-1)}.\]

(Nguyễn Văn Huyện)

Lời giải. (a) Do $k > 2$ nên $\displaystyle \frac{(k-2)(k+1)^2}{27k^3} > 0,$ khi đó thay $q = \displaystyle \frac{(k^2-1)p^2}{3k^2}$ kết hợp với
\[r \geqslant \frac{p^3(k-2)(k+1)^2}{27k^3},\]
ta được
\[\frac{a+b}{c}+\frac{b+c}{a}+\frac{c+a}{b} =\frac{pq}{r}-3  \leqslant \frac{p \cdot \frac{(k^2-1)p^2}{3k^2}}{\frac{p^3(k-2)(k+1)^2}{27k^3}}-3=\frac{6(k^2-k+1)}{(k+1)(k-2)}.\]
Đẳng thức xảy ra khi và chỉ khi $\left(1+\frac{3}{k+2}\right)a=b=c.$
(b) Tương tự do
\[0 < r \leqslant \frac{p^3(k+2)(k-1)^2}{27k^3},\]
cho nên
\[\frac{a+b}{c}+\frac{b+c}{a}+\frac{c+a}{b} =\frac{pq}{r}-3  \geqslant \frac{p \cdot \frac{(k^2-1)p^2}{3k^2}}{\frac{p^3(k+2)(k-1)^2}{27k^3}}-3 = \frac{6(k^2+k+1)}{(k+2)(k-1)}.\]
Đẳng thức xảy ra khi và chỉ khi $\left(1-\frac{3}{k+2}\right)a=b=c.$ Bài toán được chứng minh.

Nhận xét. Trường hợp $k = 3$ ta được bài toán sau

Cho ba số dương $a,\,b,\,c$ thỏa mãn điều kiện $a^2+b^2+c^2 = \frac{11}{8}(ab+bc+ca).$ Chứng minh rằng
\[\frac{39}{5} \leqslant \frac{a+b}{c}+\frac{b+c}{a}+\frac{c+a}{b} \leqslant \frac{21}{2}.\]

(Võ Quốc Bá Cẩn)

Bài 14. Với $a,\,b,\,c$ là ba số thực dương và $k$ là số thực cho trước thỏa mãn điều kiện
\[a^2+b^2+c^2 = \left (1+\frac{3}{k^2-1}  \right )(ab+bc+ca).\]
(a) Chứng minh rằng nếu $k > 2$ thì
\[\frac{a^3+b^3+c^3}{abc} \leqslant \frac{3(k^3+6k-2)}{(k-2)(k+1)^2}.\]
(b) Chứng minh rằng nếu $k > 1$ thì
\[\frac{a^3+b^3+c^3}{abc} \geqslant \frac{3(k^3+6k+2)}{(k+2)(k-1)^2}.\]

(Nguyễn Văn Huyện)
 

Lời giải. (a) Do $k > 2$ nên $\displaystyle \frac{(k-2)(k+1)^2}{27k^3} > 0,$ khi đó thay $q = \displaystyle \frac{(k^2-1)p^2}{3k^2}$ kết hợp với
\[r \geqslant \frac{p^3(k-2)(k+1)^2}{27k^3},\]
ta được
\[\frac{a^3+b^3+c^3}{abc} = \frac{p(p^2-3q)}{r}+3  \leqslant \frac{p\left[p^2-3 \cdot \frac{(k^2-1)p^2}{3k^2}\right]}{\frac{p^3(k-2)(k+1)^2}{27k^3}}+3=\frac{3(k^3+6k-2)}{(k-2)(k+1)^2}.\]
Đẳng thức xảy ra khi và chỉ khi $\left(1+\frac{3}{k+2}\right)a=b=c.$

(b) Tương tự do
\[0 < r \leqslant \frac{p^3(k+2)(k-1)^2}{27k^3},\]
cho nên
\[\frac{a^3+b^3+c^3}{abc} = \frac{p(p^2-3q)}{r}+3  \geqslant \frac{p\left[p^2-3 \cdot \frac{(k^2-1)p^2}{3k^2}\right]}{\frac{p^3(k+2)(k-1)^2}{27k^3}}+3 = \frac{3(k^3+6k+2)}{(k+2)(k-1)^2}.\]
Đẳng thức xảy ra khi và chỉ khi $\left(1-\frac{3}{k+2}\right)a=b=c.$ Chứng minh hoàn tất.

 

Bài 15. Cho ba số thực dương $a,\,b,\,c$ và $k > 1$ là số thực cho trước thỏa điều kiện
\[(a+b+c)\left(\frac{1}{a}+\frac{1}{b}+\frac{1}{c}\right) = 9+\frac{8}{k^2-1}.\]
(a) Chứng minh rằng
\[\frac{3k^2+2k+3}{(3k-1)(k+1)} \leqslant \frac{a^2+b^2+c^2}{ab+bc+ca} \leqslant \frac{3k^2-2k+3}{(3k+1)(k-1)}.\]
(b) Chứng minh rằng
\[\frac{3k^2+3k+2}{2k(k+1)} \leqslant \frac{a}{b+c}+\frac{b}{c+a}+\frac{c}{a+b} \leqslant \frac{3k^2-3k+2}{2k(k-1)}.\]
\item [(c)] Chứng minh rằng
\[\frac{2(k+1)^3+(k-1)^3}{(k-1)(k+1)^2} \leqslant \frac{a^3+b^3+c^3}{abc} \leqslant \frac{2(k-1)^3+(k+1)^3}{(k+1)(k-1)^2}.\]

(Nguyễn Văn Huyện)

Lời giải. (a) Bất đẳng thức cần chứng minh tương đương với
\begin{equation} \label{lab24}
\frac{(3k+1)^2}{(3k-1)(k+1)} \leqslant \frac{(a+b+c)^2}{ab+bc+ca} \leqslant \frac{(3k-1)^2}{(3k+1)(k-1)}.
\end{equation}
Viết giả thiết của bài toán lại dưới dạng
\[(a+b+c)(ab+bc+ca) = \frac{(9k^2-1)abc}{k^2-1},\]
hay là $r = \displaystyle \frac{pq(k^2-1)}{9k^2-1},$ thay giá trị này vào
\[p^2q^2 - 4q^3 + 2p(9q - 2p^2)r - 27r^2 \geqslant 0,\]
ta được
\[p^2q^2 - 4q^3 + 2p(9q - 2p^2)\cdot\frac{pq(k^2-1)}{9k^2-1} - \frac{27p^2q^2(k^2-1)^2}{(9k^2-1)^2} \geqslant 0,\]
hay là
\[\frac{4q\left [p^2(3k-1)(k+1)-(3k+1)^2q \right ]\left [p^2(3k+1)(k-1)-(3k-1)^2q \right ]}{(9k^2-1)^2} \leqslant 0,\]
suy ra
\[\frac{(3k+1)^2}{(3k-1)(k+1)} \leqslant \frac{p^2}{q} \leqslant \frac{(3k-1)^2}{(3k+1)(k-1)}.\]
Đẳng thức bên vế trái xảy ra khi và chỉ khi $\left(1+\frac{2}{k-1}\right)a = b = c,$ còn đẳng thức bên vế phải xảy ra khi và chỉ khi $\left(1-\frac{2}{k+1}\right)a = b = c.$

(b) Từ giả thiết ta có $r = \displaystyle \frac{pq(k^2-1)}{9k^2-1},$ do đó
\[\begin{aligned}\frac{a}{b+c}+\frac{b}{c+a}+\frac{c}{a+b} & = \frac{p^3+r}{pq-r} - 2 = \frac{p^3+r}{pq-\dfrac{pq(k^2-1)}{9k^2-1}} - 2 \\& = \frac{9k^2-1}{8k^2} \cdot \frac{p^2}{q}-\frac{15k^2+1}{8k^2}.\end{aligned}\]
Theo \eqref{lab24} thì $\displaystyle \frac{p^2}{q} \leqslant \frac{(3k-1)^2}{(3k+1)(k-1)},$ cho nên
\[\begin{aligned}\frac{a}{b+c}+\frac{b}{c+a}+\frac{c}{a+b} & \leqslant \frac{9k^2-1}{8k^2} \cdot \frac{(3k-1)^2}{(3k+1)(k-1)}-\frac{15k^2+1}{8k^2} \\& = \frac{3k^2-3k+2}{2k(k-1)}.\end{aligned}\]
Đẳng thức xảy ra khi và chỉ khi $\left(1-\frac{2}{k+1}\right)a = b = c.$ Tương tự vì
$$\displaystyle \frac{p^2}{q} \geqslant \frac{(3k+1)^2}{(3k-1)(k+1)},$$
cho nên
\[\begin{aligned} \frac{a}{b+c}+\frac{b}{c+a}+\frac{c}{a+b} & \geqslant \frac{9k^2-1}{8k^2} \cdot \frac{(3k+1)^2}{(3k-1)(k+1)}-\frac{15k^2+1}{8k^2} \\& = \frac{3k^2+3k+2}{2k(k+1)}.\end{aligned}\]
Đẳng thức xảy ra khi và chỉ khi $\left(1+\frac{2}{k-1}\right)a = b = c.$

(c) Từ giả thiết ta có $\displaystyle r = \frac{pq(k^2-1)}{9k^2-1},$ do đó
\[\begin{aligned} \frac{a^3+b^3+c^3}{abc} & = \frac{p(p^2-3q)}{r} + 3 = \frac{p(p^2-3q)}{\frac{pq(k^2-1)}{9k^2-1}} + 3 \\& = \left ( 9+\frac{8}{k^2-1} \right )\frac{p^2}{q}-\frac{24}{k^2-1}-24.\end{aligned}\]
Theo \eqref{lab24} thì $\displaystyle \frac{p^2}{q} \leqslant \frac{(3k-1)^2}{(3k+1)(k-1)},$ cho nên
\[\begin{aligned} \frac{a^3+b^3+c^3}{abc} & \leqslant \left ( 9+\frac{8}{k^2-1} \right ) \cdot \frac{(3k-1)^2}{(3k+1)(k-1)}-\frac{24}{k^2-1}-24 \\& = \frac{2(k-1)^3+(k+1)^3}{(k+1)(k-1)^2}.\end{aligned}\]
Đẳng thức xảy ra khi và chỉ khi $\left(1+\frac{2}{k-1}\right)a = b = c.$

Vì $\displaystyle \frac{p^2}{q} \geqslant \frac{(3k+1)^2}{(3k-1)(k+1)},$ cho nên
\[\begin{aligned} \frac{a^3+b^3+c^3}{abc} & \geqslant \left ( 9+\frac{8}{k^2-1} \right ) \cdot \frac{(3k+1)^2}{(3k-1)(k+1)}-\frac{24}{k^2-1}-24 \\& = \frac{2(k+1)^3+(k-1)^3}{(k-1)(k+1)^2}.\end{aligned}\]
Đẳng thức bên vế trái xảy ra khi và chỉ khi $\left(1+\frac{2}{k-1}\right)a = b = c.$ Chứng minh hoàn tất.

Nhận xét. Trường hợp $k = \sqrt{3}$ ta được bài toán sau

Cho ba số thực dương $a,\,b,\,c$ thỏa điều kiện
\[(a+b+c)\left(\frac{1}{a}+\frac{1}{b}+\frac{1}{c}\right) = 13.\]
Chứng minh rằng
\[11-2\sqrt{3} \leqslant \frac{a^3+b^3+c^3}{abc} \leqslant 11+2\sqrt{3}.\]

(Phạm Kim Hùng)

Bài 16. Cho ba số thực $a,\,b,\,c$ thỏa mãn $a+b+c=3k$ và $a^2+b^2+c^2 = 3k^2 + 6t^2$ (với $k$ bất kỳ và $t \geqslant 0$ là hai số cho trước). Chứng minh rằng
\begin{equation}
(k-2t)(k+t)^2 \leqslant abc \leqslant (k+2t)(k-t)^2.
\end{equation}

(Võ Quốc Bá Cẩn)

Lời giải. Từ giả thiết ta có $p = 3k$ và $q = 3k^2 - 3t^2,$ áp dụng vế phải của \eqref{lab23} ta được
\[abc \le \frac{3k\big[9(3k^2-3t^2)-2(3k)^2\big]+2\sqrt{\big[(3k)^2-3(3k^2-3t^2)\big]^3}}{27}=(k+2t)(k-t)^2.\]
Tương tự thì
\[abc \geqslant (k-2t)(k+t)^2.\]
Đẳng thức xảy ra khi và chỉ khi
\[(a,\,b,\,c) = \left\{(k-2t,\,k+t,\,k+t),\,(k+2t,\,k-t,\,k-t)\right\},\]
và các hoán vị. Bài toán được chứng minh.

 

Bài 17. Với $a,\,b,\,c$ là ba số thực dương thỏa mãn $a^2+b^2+c^2=3.$ Chứng minh rằng
\[\frac{a^2}{a+b}+\frac{b^2}{b+c}+\frac{c^2}{c+a}+\frac{a+b+c}{6}\geq 2.\]

(Michael Rozenberg)

Lời giải. Bởi vì
\[\sum \frac{a^2}{a+b}-\sum\frac{b^2}{a+b} = 0,\]
nên
\[\sum \frac{a^2}{a+b} = \sum\frac{b^2}{a+b},\]
do đó bất đẳng thức trên tương đương với
\[\frac{a^2+b^2}{a+b}+\frac{b^2+c^2}{b+c}+\frac{c^2+a^2}{c+a}+\frac{a+b+c}{3}\geq 4,\]
hay là
\begin{equation} \label{vidu16}
\frac{a^2+b^2}{a+b}+\frac{b^2+c^2}{b+c}+\frac{c^2+a^2}{c+a} + \frac{a+b+c}{3}\geq 4\sqrt{\frac{a^2+b^2+c^2}{3}}.
\end{equation}
Chuẩn hóa $a+b+c=3$ và đặt $a^2+b^2+c^2=3+6t^2,\,ab+bc+ca=3-3t^2$ với $0 \leqslant t < 1.$ Bất đẳng thức \eqref{vidu16} trở thành
\[13 - \frac{18(t^4-5t^2+4)}{9(1-t^2)-abc}\geqslant 4\sqrt{1+2t^2}.\]
Vì $0 \leqslant t < 1$ nên $t^4-5t^2+4 \geqslant 0$ và $abc \leqslant (1+2t)(1-t)^2,$ do đó
\[13 - \frac{18(t^4-5t^2+4)}{9(1-t^2)-abc} \geqslant 13 - \frac{18(t^4-5t^2+4)}{9(1-t^2)-(1+2t)(1-t)^2} = \frac{9t^2+4t+8}{t+2}.\]
Cuối cùng, ta sẽ chỉ ra
\[\frac{9t^2+4t+8}{t+2} \geqslant 4\sqrt{1+2t^2},\]
nhưng đây là một bất đẳng thức đúng vì
\[\left ( \frac{9t^2+4t+8}{t+2} \right )^2 - 16(1+2t^2) = \frac{t^2(7t-4)^2}{(t+2)^2} \geqslant 0.\]
Trong trường hợp tổng quát đẳng thức xảy ra khi và chỉ khi $a=b=c$ hoặc $a=5b=5c$ cùng các hoán vị. Bài toán được chứng minh.

Bài 18. Cho ba số thực dương $a,\,b,\,c$ thỏa mãn điều kiện $abc = 1.$ Chứng minh rằng
\[\frac{a+b+c}{3} \geq \sqrt[10]{\frac{a^{3}+b^{3}+c^{3}}{3}}.\]

(Michael Rozenberg)

Lời giải. Trước hết, ta có bổ đề:
Bổ đề. Cho $a, \, b, \, c$ là ba số thực dương. Chứng minh rằng
\begin{equation} \label{vidu19}
729abc(a^3+b^3+c^3+2abc) \leqslant 5(a+b+c)^6.
\end{equation}
Thật vậy, ta chuyển bổ đề về $pqr$ như sau
\[729r(p^3-3pq+5r) \leqslant 5p^6.\]
Chuẩn hóa $p = 3$ và đặt $q = 3-3t^2$ với $0 \le t < 1,$ bất đẳng thức trên trở thành
\[r(27t^2+5r) \leqslant 5.\]
Vì $r \leqslant (1+2t)(1-t)^2,$ nên
\[\begin{aligned} r(27t^2+5r) & \leqslant (1+2t)(1-t)^2[27t^2+5(1+2t)(1-t)^2] \\& =(1+2t)(1-t)^2(10t^3+12t^2+5).\end{aligned}\]
Ta chứng minh
\[(1+2t)(1-t)^2(10t^3+12t^2+5) \leqslant 5,\]
tương đương với
\[t^2(3-20t+36t^2+6t^3-20t^4) \geqslant 0,\]
hay là
\[t^2[3(2t-1)^4+4t(1-t)(4t-1)^2+2t^3(3-2t)] \geqslant 0.\]
Như vậy \eqref{vidu19} được trước minh. Quay trở lại bài toán, từ \eqref{vidu19} với điều kiện $abc=1,$ ta được
\[\frac{a+b+c}{3} \geqslant \sqrt[6]{\frac{a^3+b^3+c^3+2}{5}}.\]
Áp dụng bất đẳng thức AM-GM, ta có
\[a^3+b^3+c^3+2 = 3\cdot\frac{a^3+b^3+c^3}{3}+1+1 \geqslant 5\sqrt[5]{\frac{(a^3+b^3+c^3)^3}{27}},\]
suy ra
\[\frac{a+b+c}{3} \geq \sqrt[10]{\frac{a^{3}+b^{3}+c^{3}}{3}}.\]
Đẳng thức xảy ra khi và chỉ khi $a=b=c=1.$ Bài toán được chứng minh.

 

Như đã trình bày, đây là một hướng phát triển khác của kỹ thuật $pqr,$ tuy khá cồng kềnh về mặt hình thức, nhưng mặt xử lý lại rất trực quan, không mang tính đánh đố mà chỉ đòi hỏi khả năng xử lý, tính toán, đồng thời thông qua các ví dụ rất khó và phức tạp trên, ta phần nào thấy được tính hiệu quả của nó. Những kết quả trong bài viết mang tính khái quát nên rất chặt, thông thường mức độ của các bất đẳng thức trong các kỳ thi chưa đến mức như vậy, điển hình là bài $3$ của kỳ thi IMO $2006$ (lời giải gọn gàng hơn rất nhiều so với đáp án chính thức), nên kỹ thuật này cũng có thể xem như một lựa chọn khi gặp lớp các bất đẳng thức ba biến hoán vị.

 

4. Bài tập rèn luyện

Bài tập 1. Cho ba số thực không âm $a,\,b,\,c$ thỏa mãn $ab+bc+ca > 0.$ Chứng minh rằng
\[\frac{a^2+b^2+c^2}{ab+bc+ca} \geqslant \frac{3(a^2+b^2+c^2)}{(a+b+c)^2} + \frac{\big(1+\sqrt{2}\big)^2(a-b)^2(b-c)^2(c-a)^2}{(a+b)^2(b+c)^2(c+a)^2}.\]

(Michael Rozenberg, Nguyễn Văn Huyện)

Bài tập 2. Chứng minh rằng bất đẳng thức
\[a^2+b^2+c^2-ab-bc-ca \geqslant \sqrt{9+6\sqrt{3}}\left|\frac{a^3-b^3}{a+b}+\frac{b^3-c^3}{b+c}+\frac{c^3-a^3}{c+a}\right|,\]
luôn đúng với mọi số thực không âm $a,\,b,\,c$ thỏa mãn điều kiện $ab+bc+ca>0.$

(Nguyễn Văn Huyện, VMEO IV)

Bài tập 3. Cho ba số thực $a,\,b,\,c$ thỏa mãn điều kiện $a^2+b^2+c^2 > 0$ và
\[a^2+b^2+c^2=5(ab+bc+ca).\]
Chứng minh rằng
\[-\frac{1}{25} \leqslant \frac{a^3b+b^3c+c^3a}{(a^2+b^2+c^2)^2} \leqslant \frac{197}{675}.\]
(Vasile Cîrtoaje)

Bài tập 4. Cho ba số thực không âm $x,\,y,\,z$ thỏa mãn $xy+yz+zx>0.$ Chứng minh rằng
\[\frac{x}{y+z}+\frac {y}{z+x}+\frac {z}{x+y} \geqslant \frac{3}{2}+\frac{\sqrt{13+
16\sqrt{2}}}{2}\left(\frac{x-y}{x+y}+\frac{y-z}{y+z}+\frac{z-
x}{z+x}\right).\]

(Liu Quan Bao)

Bài tập 5. Cho ba số thực dương $a,\,b,\,c$ và số thực $k > 1$ cho trước thỏa mãn điều kiện
\[(a+b+c)\left(\frac{1}{a}+\frac{1}{b}+\frac{1}{c}\right) = 9 + \frac{8}{k^2-1}.\]
Chứng minh rằng
\[\frac{2(k+1)^4+(k-1)^2(k^2+1)}{2(k^2+1)(k+1)^2} \leqslant \frac {a^2}{b^2+c^2}+\frac {b^2}{c^2+a^2}+\frac {c^2}{a^2+b^2} \leqslant \frac {2(k-1)^4+(k^2+1)(k+1)^2}{2(k-1)^2(k^2+1)}.\]

(Nguyễn Văn Huyện)

Bài tập 6. Cho ba số thực không âm $a,\,b,\,c$ thỏa mãn $a^2+b^2+c^2=3.$ Chứng minh rằng
\[a^2b+b^2c+c^2a \leqslant \frac{1}{2}abc(a+b+c-1)+2.\]

(Võ Quốc Bá Cẩn)

 

 

Bài tập 7. Với $a,\,b,\,c$ là ba số thực không âm. Chứng minh rằng với mọi số thực dương $k \geqslant 2$ cho trước sao cho
\[a^2+b^2+c^2 = \frac{k^2+1}{k}(ab+bc+ca).\]
Chứng minh rằng
\[a^3b+b^3c+c^3a \leqslant \frac{k^3}{(k^2+1)^2}(a^2+b^2+c^2)^2.\]

(Nguyễn Văn Huyện)

Bài tập 8. Cho $a,\,b,\,c$ là ba số thực dương. Chứng minh rằng
\[(a+b+c)\left(\frac{1}{a}+\frac{1}{b}+\frac{1}{c}\right) \geqslant 9\sqrt{\frac{a^2+b^2+c^2}{ab+bc+ca}}.\]

(Nguyễn Văn Huyện)

Bài tập 9. Cho $a,\,b,\,c$ là ba số thực không âm. Chứng minh rằng
\[a^4+b^4+c^4-abc(a+b+c) \geqslant 2\sqrt{2}\left|a^3b+b^3c+c^2a-ab^3-bc^3-ca^3\right|.\]

(Phạm Kim Hùng)

Bài tập 10. Với $a,\,b,\,c$ là ba số thực dương và số thực $k \geqslant 0$ cho trước thỏa mãn điều kiện
\[(a+b+c)\left ( \frac{1}{a}+\frac{1}{b}+\frac{1}{c} \right )=(k+3)^2.\]
Tìm giá trị lớn nhất và giá trị nhỏ nhất của biểu thức
\[P = \frac{a}{b}+\frac{b}{c}+\frac{c}{a}.\]

(Nguyễn Văn Huyện)

5. Tài liệu thảm khảo

  1. Võ Quốc Bá Cẩn, Chuyên Đề Bất Đẳng Thức Hiện Đại, 2008.
     
  2. Vasile Cîrtoaje, Algebraic Inequalities Polynomial Rational Symmetric Inequalities, GIL Publishing House, 2011.
     
  3. Vasile Cîrtoaje, Võ Quốc Bá Cẩn , On Some Cyclic Homogeneous Polynomial Inequality Of Degree Fourth In Real Variables Under Constraints, International Journal of Pure and Applied Mathematics, 2012.
     
  4. Art of Problem Solving: http://artofproblemsolving.com

File gửi kèm


Bài viết đã được chỉnh sửa nội dung bởi Nguyenhuyen_AG: 27-07-2016 - 23:27

Nguyen Van Huyen
Ho Chi Minh City University Of Transport

#4
cyndaquil

cyndaquil

    Hạ sĩ

  • Thành viên
  • 63 Bài viết

2. Bất đẳng thức hoán vị

Sang phần này, ta sẽ tiếp tục khai thác đại lượng $P$ trong việc xử lý các bất đẳng thức hoán vị. Đây là một hướng phát triển khác của $pqr$ được gọi là “$pqr$ hoán vị” và được thực hiện như sau. Biến đổi khéo léo các biểu thức hoán vị để làm xuất hiện $Q=(a-b)(b-c)(c-a)$ cũng là một đại lương hoán vị. Tuy nhiên, ta hoàn toàn có thể biễu diễn $Q$ dưới dạng $pqr$ thông qua đánh giá $Q \leqslant |Q| = \sqrt{P}$ đến đây, sử dụng \eqref{lab0} và ta giải tiếp bằng $pqr.$ Lưu ý rằng, ngoài các quan hệ sẵn có, bản thân bất đẳng thức $P \geqslant 0$ cũng đã cho ta một đánh giá tương quan giữa $p,\,q$ và $r.$

 

Cho em hỏi bài này dùng "pqr hoán vị" thì làm tnao ạ

Cho các số thực dương $a,b,c$ có tổng bằng 3

Chứng minh $\frac ab+\frac bc+\frac ca+9abc \ge 9$


Bài viết đã được chỉnh sửa nội dung bởi cyndaquil: 27-07-2016 - 23:03


#5
royal1534

royal1534

    Trung úy

  • Điều hành viên THCS
  • 773 Bài viết

Cái đầu tiên là "thành viên VMF" anh ơi :V






0 người đang xem chủ đề

0 thành viên, 0 khách, 0 thành viên ẩn danh